LSAT and Law School Admissions Forum

Get expert LSAT preparation and law school admissions advice from PowerScore Test Preparation.

 Administrator
PowerScore Staff
  • PowerScore Staff
  • Posts: 8916
  • Joined: Feb 02, 2011
|
#81498
Complete Question Explanation

Strengthen, Principle. The correct answer choice is (E).

Answer choice (A):

Answer choice (B):

Answer choice (C):

Answer choice (D):

Answer choice (E): This is the correct answer choice.

This explanation is still in progress. Please post any questions below!
 kmpaez
  • Posts: 12
  • Joined: Sep 18, 2017
|
#42857
I correctly chose answer choice E, but do not fully understand why answer choices A and C are incorrect. Any clarification would be appreciated.
 James Finch
PowerScore Staff
  • PowerScore Staff
  • Posts: 943
  • Joined: Sep 06, 2017
|
#42990
Hi KM,

The stimulus gives us two drugs, Y and Z, that treat heart attacks. Y definitively does not treat heart attacks better than Z, and Z treats heart attacks at least as well as Y, and possibly better. However, Z is more expensive than Y. The information about Z's marketing is a red herring. Question 18 asks us to justify a conclusion to always use drug Z over drug Y.

Answer choice (A) is an attractive wrong answer, but fails us in that it gives us another conditional relationship, one not present in the stimulus, about patient finances and treatment. We are concerned with cost,irrespective of who pays that cost, and effectiveness of treatment. Moreover, we are looking for a principle that would justify always using Z over Y when that option is present, not one that justifies that treatment under certain circumstances.

Answer choice (B) is irrelevant. The only thing mentioned in the stimulus relating to doctor compensation is the drug Z marketing scheme, which as mentioned above is a red herring.

Answer choice (C) is also irrelevant. The stimulus's argument is concerned with medical effectiveness versus cost, not shady marketing practices. Still a red herring.

Answer choice (D) clearly irrelevant. Nobody is even criticizing the studies.

Answer choice (E) finally justifies using Z over Y by making Z's drawback, cost, irrelevant and making even the possibility of being more effective determinant in which treatment to use.

Hope this helps!
 SammyWu11201
  • Posts: 29
  • Joined: Jun 29, 2020
|
#82586
Hey! I don't understand your explanation of E. The conclusion tells us to take drug Y over drug Z. The conclusion also somewhat rules out the possibility of Z being more effective than Y, so it would be tied with Y because Y can't be more effective than Z. Still, the conclusion is telling us to take Z over Y. I just don't understand how E would be the right answer. The sufficient condition for E says "Where alternative treatments exist and there is a chance that one is more effective than the other..." Well, we already know that there is no one that is more effective than the other one.

Am I missing something here?
User avatar
 KelseyWoods
PowerScore Staff
  • PowerScore Staff
  • Posts: 1079
  • Joined: Jun 26, 2013
|
#82851
Hi Sammy!

According to the stimulus, there is a chance that Z is more effective than Y: "drug Y does this no more effectively than the more expensive drug Z, but drug Z is either no more or only slightly more effective than drug Y." The author says that "drug Z does not clearly treat the problem more effectively than drug Y" but that doesn't mean there's not a chance that Z is more effective. Basically the author is saying there's no proof that Z is better than Y so therefore there is no reason to use Z instead of Y. But even though there is no proof that Z is better than Y, there is also no proof that it isn't better than Y. The studies tell us that Y is definitely not more effective than Z, but they leave open the possibility that Z is slightly more effective than Y. That means there is still a chance that Z is slightly more effective than Y. Thus, the principle in answer choice (E) would justify using Z over Y because there is still a slight chance that Z is more effective.

Hope this helps!

Best,
Kelsey
 nickp18
  • Posts: 20
  • Joined: May 26, 2020
|
#95919
Hi PS,

In an earlier explanation, it was stated that answer choice "C" had to do with shady marketing practices. Is this really what C is referring to though? The way I interpreted C and the reason I selected it was that, in my mind, "if two drugs are virtually equally effective, there must be another reason as to why one is better than the other if it is being prescribed more. This might be due to one having harmful side effects, protection against other illnesses/symptoms, etc."

I guess I just fail to see how this is a red herring connecting to any sort of shady medical practices.

Thanks,

Nick
 Adam Tyson
PowerScore Staff
  • PowerScore Staff
  • Posts: 5153
  • Joined: Apr 14, 2011
|
#96179
The stimulus told us that the company that makes Z uses "questionable marketing practices," Nick. Answer C is saying the doctor shouldn't be influenced by those practices. In other words, they shouldn't let those shady practices bias them against Z. Answer C is sort of like saying "it's okay to prescribe a drug even though the maker did some questionable stuff to promote it; their questionable marketing practices don't make it a bad choice."

But that doesn't mean that they SHOULD prescribe Z, either, and we need an answer that more actively supports their decision to prescribe Z, like answer E does. We know that Z COULD be more effective than Y, and answer E tells us that the possibility of being more effective means you have to go with that option.

It looks to me like you were trying to help answer C be better by making some assumptions, including adding a fact not given in the stimulus. Nobody said Z is prescribed more than Y is! And even if it is, we should not assume that's due to a valid medical reason, because there could be many reasons for that difference, if such a difference actually existed. Don't make assumptions, and don't try to help answers be better than they are! The right answer will be right without any help from us!

Get the most out of your LSAT Prep Plus subscription.

Analyze and track your performance with our Testing and Analytics Package.